Which one of the following CANNOT be true?

JadeKassab on November 21, 2022

I do not understand why G cannot be presented before M.

When I watched the real time video, it didn't help me to understand why this could not work. When I made a big chain it looked like this: U-G-M-o &P Then I placed them into 1,2,3 and put U at 1pm in E and G at 1 pm in W. Then I was able to place MOP and S. What am I missing? Why does this not work. When I tested each answer, they all worked. So I was not able to use process of elimination on this one.

Reply
Create a free account to read and take part in forum discussions.

Already have an account? log in

Emil-Kunkin on November 22, 2022

This is impossible since the earliest M could go is at 2 pm. There are two things that must go after M, so M is either at 1 or 2. If G is before M, then G must be in 1. However, our last rule tells us G cannot be in 1, since G is after U. U and G cannot both be at 1.